Một số định lý trong lý thuyết tập hợp cực trị

Những bài toán trong lý thuyết tập hợp cực trị thường có dạng: cho một

tập (hay 1 họ các tập) S thỏa mãn một số điều kiện cho trước, tìm max

hoặc min của |S|. Khi nào dấu đẳng thức xảy ra?

Ta xét một bài tập đơn giản

pdf7 trang | Chia sẻ: hainam | Lượt xem: 2232 | Lượt tải: 0download
Bạn đang xem nội dung tài liệu Một số định lý trong lý thuyết tập hợp cực trị, để tải tài liệu về máy bạn click vào nút TẢI VỀ ở trên
Một số định lý trong lý thuyết tập hợp cực trị
Vũ Thế Khôi, email: vtkhoi@math.ac.vn
Viện Toán Học, 18 Hoàng Quốc Việt, Hà Nội
1 Giới thiệu
Những bài toán trong lý thuyết tập hợp cực trị thường có dạng: cho một
tập (hay 1 họ các tập) S thỏa mãn một số điều kiện cho trước, tìm max
hoặc min của |S|. Khi nào dấu đẳng thức xảy ra?
Ta xét một bài tập đơn giản
Bài toán 1. Cho F là một họ các tập con của một tập n phần tử X. Giả sử
bất kỳ hai phần tử nào của F cũng có giao khác rỗng. Hỏi giá trị lớn nhất
có thể của |F| là bao nhiêu?
Bài toán 1’. Cho F là một họ các tập con của một tập n phần tử X. Giả
sử bất kỳ hai phần tử A,B nào của F cũng có A∪B 6= X . giá trị lớn nhất
có thể của |F| là bao nhiêu?
Bài toán 2. Cho F là một họ các tập con của một tập n phần tử X. Giả
sử bất kỳ hai phần tử A,B nào của F cũng có A ∩ B 6= ∅ và A ∪ B 6= X .
giá trị lớn nhất có thể của |F| là bao nhiêu?
2 Các định lý cơ bản
Ta gọi một tập có k phần tử là k-tập. Một họ F các tập con của một n-tập
được gọi là họ giao nếu với mọi A,B ∈ F ta có A ∩B 6= ∅.
Định lý Erdo¨s-Ko-Rado. Một họ giao F các k-tập con của một n-tập
X(k ≤ n/2) có không quá (n−1k−1) phần tử.
Chứng minh. Đặt X := {0, 1, 2, . . . , n − 1}. Với s ∈ X, ký hiệu Bs :=
{s, s + 1, . . . , s+ k − 1}. trong đó các số mod n.
1
Ta nhận thấy trong số các tập Bs, Có không quá k tập nằm trong F . Thật
vậy, giả sửB0 ∈ F . Như vậy có đúng 2k−2 tập khác làBi,−(k−1) ≤ i ≤ k−1
có giao khác rỗng với B0. Tuy nhiên các cặp Bi và Bi+k lại rời nhau nên
ngoài B0 ra, F chỉ chứa nhiều nhất là k − 1 tập Bs.
Với σ là một hoán vị của X, ta đặt σ(Bs) := {σ(s), σ(s + 1), . . . , σ(s +
k − 1)}, ký hiệu L := {(σ, s)|σ(Bs) ∈ F}. Từ nhận xét ngay trên ta có
|L| ≤ k(n!). Mặt khác với mỗi s và phần tử A ∈ F cố định, có đúng
k!(n − k)! hoán vị σ mà σ(Bs) = A. Do đó |L| = n|F|k!(n − k)!. Như vậy
|F| ≤ kn!
nk!(n− k)! =
(
n− 1
k − 1
)
.
Định lý Sperner. Cho F là một họ các tập con của một n-tập X thỏa
mãn A * B với mọi A,B ∈ F . Ta có |F| ≤ ( nbn
2
c
)
.
Chứng minh Ta nhận thấy số các dãy tăng các tập con
M1 ⊂M2 ⊂ · · · ⊂Mn−1 ⊂Mn = X với |Mi| = i đúng bằng n!.
Nếu A là một r-tập thì số các dãy tăng các tập con
M1 ⊂ · · · ⊂Mr−1 ⊂ A ⊂Mr+1 ⊂ · · · ⊂Mn = X với |Mi| = i đúng bằng
r!(n− r)!.
Ta nhận thấy nếu A,B ∈ F thì tập các dãy tăng chứa A và tập các dãy
tăng chứa B là rời nhau. Như vậy nếu F = {A1, A2, . . . , Ap} với |Ai| = ni
thì ta có :
p∑
i=1
ni!(n − ni)! ≤ n!.
Như vậy
p( n
bn
2
c
) ≤
p∑
i=1
1( n
ni
) ≤ 1.
Với F là họ tất cả các bn
2
c-tập con thì hiển nhiên đẳng thức xảy ra.
Từ chứng minh trên ta nhận được ngay hệ quả sau.
Hệ quả.(Bất đẳng thức Lubell-Yamamoto-Meshalkin) Cho F là một họ
các tập con của một n-tập X thỏa mãn A * B với mọi A,B ∈ F . Gọi ak là
2
số các k-tập con thuộc F . Khi đó
n∑
k=0
ak(n
k
) ≤ 1.
Định lý De Bruijn Erdo¨s. Cho F là một họ các tập con của một
n-tập X thỏa mãn:
i) |F| ≥ 2, và |A| ≥ 2,∀A ∈ F .
ii) Với 2 phần tử bất kỳ x, y ∈ X tồn tại duy nhất A ∈ F với x, y ∈ A.
Khi đó |F| ≥ n.
Chứng minh. Đặt |F| = m và rx := |{A ∈ F|x ∈ A}| với x ∈ X. Nếu
x /∈ A, dễ thấy rx ≥ |A| vì bất kỳ y ∈ A cũng có một tập B chứa x và y.
Giả sử m ≤ n, khi đó ta có m(n− |A|) ≥ n(m− rx) với mọi x ∈ A. Ta có
1 =
∑
A∈F
∑
x/∈A
1
m(n− |A|) ≤
∑
x∈X
∑
A:x/∈A
1
n(m− rx) = 1.
Do đó dấu đẳng thức xảy ra ở mọi chỗ, tức là ta có m ≥ n và nếu m = n
thì |A ∩B| = 1,∀A,B ∈ F .
3 Một số bài tập về tập hợp
♦ Cho k ≥ 1 là một số tự nhiên. Tìm số tự nhiên nhỏ nhất n sao cho với
mọi tập gồm n số nguyên luôn có 2 số mà tổng hoặc hiệu của chúng chia
hết cho 2k + 1.
Gợi ý: tập {k + 1, . . . , 2k + 1} không thỏa mãn tính chất của bài toán,
vậy n > k + 1. Ta chứng tỏ n = k + 2. Với một tập A bất kỳ gồm k + 2 số,
xét tập A′ các phần dư mod 2k + 1 của A như một tập con của {−k,−k +
1, . . . , 0, 1, · · · , k}. Nếu |A′| = k+ 2 thì luôn có hai phần tử có tổng bằng 0.
♦ Cho (Ai)ni=1, (Bj)nj=1, (Ck)nk=1 là ba phân hoạch của một tập hữu hạn X.
giả sử với mọi bộ ba số (i, j, k) ta có :
|Ai ∩Bj|+ |Bj ∩ Ck|+ |Ck ∩Bi| ≥ n.
3
Chứng tỏ rằng |X| ≥ n3/3. Với |X| = n3/3, hãy chỉ ra các phân hoạch để
dấu đẳng thức xảy ra.
Gợi ý: Cộng tất cả các BĐT, ta được điều cần cm. Nếu |X| = n3/3, viết
X = E11 ∪ E12 ∪ . . . ∪ E1n ∪ E21 ∪ E22 ∪ . . . ∪ E2n ∪ . . . ∪ En1 ∪ En2 ∪ . . . ∪Enn .
với |Eji | = n/3,∀i, j. Đặt Ai := ∪nj=1Aij , Bj := ∪ni=1Aij, Ck := ∪ni=1Aik+i−1≡n
ta được các phân hoạch mà dấu đẳng thức xảy ra.
♦ Xác định số n lớn nhất sao cho tồn tại các tập phân biệt S1, S2, . . . , Sn
thỏa mãn:
i) |Si ∪ Sj| ≤ 2006 với mọi 1 ≤ i, j ≤ n.
ii) Si ∪ Sj ∪ Sk = {1, 2, . . . , 2010} với mọi 1 ≤ i < j < k ≤ n.
Gợi ý: Đặt G{i,j} := {1, 2, . . . , 2010} \ (Si ∪ Sj) với 1 ≤ i, j ≤ n. Từ (i)
ta có |G{i,j}| ≥ 4 còn từ (ii) ta có G{i,j} ∩ G{k,l} = ∅ nếu i 6= j, k 6= l và
{i, j} 6= {k, l}. Từ đó ta có
2010 ≥ | ∪1≤i<j≤n G{i,j}| =
∑
1≤i<j≤n
|G{i,j}| ≥ 4
(
n
2
)
.
Do đó n ≤ 32.
Ví dụ với n = 32 xây dựng như sau: chia tập {1, 2, . . . , 1984} thành 496
4-tập rời nhau G{i,j}, 1 ≤ i < j ≤ 32. Xác định tập Si := {1, 2, . . . , 2010} \
∪j 6=iG{i,j} thỏa mãn các điều kiện (i) và (ii).
♦ Cho Dk, 0 ≤ k ≤ n − 1 là tập gồm các tập con của {1, 2, . . . , n} sao cho
với bất kỳ A,B ∈ Dk ta có |A ∩B| ≤ k. Chứng minh rằng
|Dk| ≤
(
n
0
)
+
(
n
1
)
+
(
n
2
)
+ · · · +
(
n
k + 1
)
.
Gợi ý: gọi Pk là tập gồm các tập con của {1, 2, . . . , n} chứa không quá
k phần tử. Bất kỳ tập Dk cực đại nào cũng chứa Pk. Với A ∈ Dk \ Pk đặt
A˜ := {A1, A2, . . . , As|Ai ⊆ A, |Ai| = k + 1}. Như vậy họ (A˜)A∈Dk\Pk gồm
toàn các tập rời nhau. Vậy |Dk \ Pk| ≤ |Pk+1 \ Pk| hay |Dk| ≤ |Pk+1|.
♦ (Chọn đội tuyển Romania 1999) Cho A là một tập có n phần tử. Giả sử
A1, A2, . . . , Am là các tập con thỏa mãn |Ai| = 3,∀i và |Ai∩Aj | ≤ 1,∀i 6= j.
4
Chứng minh rằng tồn tại một tập con X của A không chứa bất kỳ tập Ai
nào và |X| ≥ b√2nc.
Gợi ý: giả sử X là một tập con tối đại thỏa mãn đầu bài. Khi đó với
mỗi u ∈ A \X sẽ có 2 phần tử x1, x2 ∈ X sao cho tập {u, x1, x2} trùng với
một tập Ai nào đó. Theo đầu bài thì đây là một đơn ánh từ A \X đến tập
gồm các 2-tập của X. Vậy ta có
n− |X| ≤ |X|(|X| − 1)/2.
Từ đây ta có điều cần chứng minh.
♦ Giả sử X là một tập hữu hạn và Ei, 1 ≤ i ≤ m là các tập con thỏa mãn
|Ei| ≥ 2,∀i và |Ei ∩ Ej | 6= 1,∀i, j. Chứng minh rằng có thể tô tập X bằng
hai mầu sao cho mỗi tập Ei đều chứa các phần tử được tô cả hai mầu.
Gợi ý: ĐặtX = {x1, x2, . . . , xn}.Giả sử ta đã tô màuXk := {x1, x2, . . . , xk}
bằng hai màu xanh và đỏ thỏa mãn đầu bài nhưng nếu thêm xk+1 thì
không tô được nữa. Khi đó tồn tại tập Ej , El chứa xk+1 và Ej, El ⊆
{x1, x2, . . . , xk+1} sao cho Ej ∩ Xk gồm toàn các màu xanh, El ∩ Xk gồm
toàn các màu đỏ. Như vậy |Ej ∩ El| = 1 mâu thuẫn với giả thuyết.
♦ (Bollobas) Cho F = {A1, A2, . . . , Am} là một họ giao các tập con của một
n-tập X với |Ai| = ni ≤ n/2. Chứng minh rằng
m∑
k=1
1(n−1
ni−1
) ≤ 1.
Gợi ý: Tương tự như trong chứng minh định lý Erdos-Ko-Rado, ta đặt
gọi L là tập các bộ (σ, s, i) mà Ai = {σ(s), σ(s + 1), . . . , σ(s + ni − 1)}. Ta
nhận được bất đẳng thức khi đếm các phần tử của L với trọng của (σ, s, i)
là 1ni .
♦ Cho F là một họ các tập con của X với |F| = m. Chứng tỏ rằng trong
số những tập có dạng (A \B)∪ (B \A) với A,B ∈ F có ít nhất m tập phân
biệt.
Gợi ý: Ánh xạ f(S) = (A \ S) ∪ (S \A) là 1− 1.
5
♦ (Bulgari 2004) Cho A1, A2, . . . , An là các tập hữu hạn thỏa mãn |Ai ∩
Ai+1| > n−2n−1 |Ai+1| với i = 1, 2, . . . , n (An+1 ≡ A1). Chứng minh rằng
∩ni=1Ai 6= ∅.
Gợi ý: Giả sử |A1 là lớn nhất. Ký hiệu Bi := Ai ∩Ai+1. Ta có :
|An| ≥ |Bn−1 ∪Bn| > n− 2
n− 1 |An|+
n− 2
n− 1 |A1| − |Bn−1 ∩Bn|.
Do đó |An−1 ∩ An ∩ A1| = |Bn−1 ∩ Bn| > n−3n−1 |A1|. Từ đó chứng minh quy
nạp rằng :
|An−k ∩An−k+1 ∩ · · · ∩A1| > n− k − 2
n− 1 |A1|.
Với k = n− 2 ta có điều cần chứng minh
♦ (Iran 1999) Giả sử X = {1, 2, . . . , n} và A1, A2, . . . , Ak là các tập con của
X sao cho với mọi 1 ≤ i1, i2, i3, i4 ≤ n ta có:
|Ai1 ∪Ai2 ∪Ai3 ∪Ai4 | ≤ n− 2.
Chứng minh rằng k ≤ 2n−2.
Gợi ý: Gọi A là tập có |A| nhỏ nhất trong số các tập thỏa mãn A * Ai ∪Aj
với i, j nào đó (không nhất thiết phân biệt). Dễ dàng chứng minh được họ
các tập S1 := {A ∩ A1, A ∩ A2, . . . , A ∩ Ak} chứa không quá 2|A|−1 phần
tử vì nếu M ∈ S1 thì A \ M /∈ S1. Tương tự ta chứng minh được tập
S2 := {B ∩ A1, B ∩ A2, . . . , B ∩ Ak}, với B = X \ A, có không quá 2|B|−1
phần tử vì nếu M ∈ S2 thì B \M /∈ S2 (trái lại thì B ⊆ Ak ∪Al, mặt khác
ta có A \m ⊆ Ai ∪Aj dẫn đến mâu thuẫn |Ai ∪Aj ∪Ak ∪Al| ≥ n− 1).
♦ Cho 35 tập A1, A2, . . . , A35 mỗi tập có 27 phần tử. Biết rằng các tập này
thỏa mãn |Ai ∩Aj ∩Ak| = 1 với mọi 1 ≤ 1 < j < k ≤ 35. Chứng minh rằng
∩35i=1Ai 6= ∅.
Gợi ý: Nếu ∃x ∈ A1 sao cho ∩35i=1Ai = {x} thì bài toán đúng. Trái lại,
xét họ các tập Bx := {Ai| x ∈ Ai}x∈A1,i=2,3,...,35 sao cho |Bx| ≥ 2. Đây
là một họ các tập con của tập A2, A3, . . . , A35 mâu thuẫn với định lý De
Bruijn-Erdo¨s.
6
♦ (Putnam 1980) Cho A1, A2, . . . , A1066 là các tập con khác nhau của một
tập hữu hạn X thỏa mãn |Ai| > |X|/2,∀i. Chứng tỏ rằng tồn tại 10 phần
tử của X sao cho mỗi tập Ai, 1 ≤ i ≤ 1066 chứa ít nhất một phần tử này.
Gợi ý: Từ giả thiết ta nhận thấy tồn tại x1 ∈ X nằm trong nhiều hơn 533
tập Ai. Gọi B1, B2, . . . , Bs, s ≤ 532 là các tập không chứa x1. Ta đưa bài
toán về bài toán tương tự với tập Y = X \{x1} và họ các tập B1, B2, . . . , Bs.
Lập luận tương tự ta tìm được x2 nằm trong nhiều hơn s/2 tập và lại đưa bài
toán về trường hợp Z = X \ {x1, x2} và họ các tập B1, B2, . . . , Bt, t ≤ 265.
Cư tiếp tục đến phần tử x10 thì nhận được dãy cần tìm.
7

File đính kèm:

  • pdfextremal_set_theory.pdf
Bài giảng liên quan